Linear Dependence/Linear Combination Question

Click For Summary
The discussion revolves around determining the linear dependence of the set S={v1, v2, v3} where v1=[1, -1], v2=[-2, 2], and v3=[3, a]. The set is linearly dependent when a=-3, as this creates a situation where v3 can be expressed as a linear combination of v1 and v2. The matrix reduction shows that the equations formed indicate a free variable, confirming the dependence. Additionally, it is noted that v2 is a scalar multiple of v1, which is relevant for understanding the relationship between the vectors. Overall, the key focus is on finding the specific value of a that satisfies these conditions.
Tim 1234
Messages
10
Reaction score
0
Member warned about posting without the homework template
S={v1, v2, v3}

v1=[1, -1], v2=[-2, 2], v3=[3, a]

a) For what value(s) a is the set S linearly dependent?
b)For what value(s) a can v3 be expressed as a linear combination of v1 and v2?

a) p=3 and m=2
3-2=1 free variable
Therefore the set has non-trivial solutions and is linearly dependent

b) I reduced the matrix to the following:

1 -2 3 0
0 0 a+3 0

Does a need to equal -3 for a linear combination to be valid?

I recognize v2=(-2)v1 - is this relevant at all?
 
Last edited by a moderator:
Physics news on Phys.org
Tim 1234 said:
S={v1, v2, v3}

v1=[1, -1], v2=[-2, 2], v3=[3, a]

a) For what value(s) a is the set S linearly dependent?
b)For what value(s) a can v3 be expressed as a linear combination of v1 and v2?

a) p=3 and m=2
3-2=1 free variable
Therefore the set has non-trivial solutions and is linearly dependent
You haven't answered the question -- "For what value(s) of a is the set linearly dependent?" Your answer should say something about a.
Tim 1234 said:
b) I reduced the matrix to the following:

1 -2 3 0
0 0 a+3 0

Does a need to equal -3 for a linear combination to be valid?

I recognize v2=(-2)v1 - is this relevant at all?
Yes, it's relevant to both question parts. Since v2 is a multiple of v1 (and vice versa), part b boils down to the question, "what values of a make v3 a scalar multiple of either v1 or v2?
 
Tim 1234 said:
S={v1, v2, v3}

v1=[1, -1], v2=[-2, 2], v3=[3, a]

a) For what value(s) a is the set S linearly dependent?
b)For what value(s) a can v3 be expressed as a linear combination of v1 and v2?

a) p=3 and m=2
3-2=1 free variable
Therefore the set has non-trivial solutions and is linearly dependent
In what sense does a set have "solutions" at all?
What I think you meant to say, and what you should say, is "this set is linearly independent if and only if the only values of p, q, and r, that make p[1, -1]+ q[-2, 2]+ r[3, a]= [0, 0] are p= q= c= 0." That gives the two equations p- 2q+ 3r= 0 and -p+ 2q+ 3r= 0. But p= 2, q= 1, r= 0 will work. Therefore the set is not independent.

b) I reduced the matrix to the following:

1 -2 3 0
0 0 a+3 0

Does a need to equal -3 for a linear combination to be valid?

I recognize v2=(-2)v1 - is this relevant at all?
Personally, I dislike changing everything to matrices (and you shouldn't say "I reduced the matrix" when you haven't yet shown a matrix to begin with!). v_3 is a linear combination of v_2 and v_3 if and only if there exist non-zero values, p and q, such that [3, a]= p[1, -1]+ q[-2, 2] which gives the two equations p- 2q= 3 and -p+ 2q= a. What happens if you add those two equations?
 
Question: A clock's minute hand has length 4 and its hour hand has length 3. What is the distance between the tips at the moment when it is increasing most rapidly?(Putnam Exam Question) Answer: Making assumption that both the hands moves at constant angular velocities, the answer is ## \sqrt{7} .## But don't you think this assumption is somewhat doubtful and wrong?

Similar threads

Replies
4
Views
2K
Replies
12
Views
2K
  • · Replies 5 ·
Replies
5
Views
5K
Replies
1
Views
2K
  • · Replies 3 ·
Replies
3
Views
1K
  • · Replies 2 ·
Replies
2
Views
2K
Replies
1
Views
2K
  • · Replies 1 ·
Replies
1
Views
2K
  • · Replies 7 ·
Replies
7
Views
3K
  • · Replies 4 ·
Replies
4
Views
2K